Está en la página 1de 32

Logical

Reasoning

PDF compression, OCR, web optimization using a watermarked evaluation copy of CVISION PDFCompressor

Assumption Questions
Directions: The questions in this section are based on the reasoning contained in brief statements or passages. For some
questions, more than one of the choices could conceivably answer the question. However, you are to choose the best answer; that
is, the response that most accurately and completely ans\.rr'ers the question. You should not make assumptions that are by
commonsense standards implausible, superfluous, or incompatible with the passage.

Roberl

Speed limits on residential streets in Crownsbury


are routinely ignored by drivers. People crossing
those streets are endangered by speeding drivers,
yet the city does not have enough police officers

Sheila's response depends on the presupposition that

Finnish author laakko Mikkeli was accused by


Norwegian author Kirsten Halden of plagiarizing a
book that she had written and that had been
published 20 years before Mikkeli's. The two books,
although set in different periods and regions, contain
enough plot similarities to make coincidental
resemblance unlikely. Mikkeli's defense rests on his
argument that plagiarism was impossible in this case
because Halden's book has been published only in
Norwegian, a language Mikkeli does not understand,
and because no reviews of Halden's book have ever
been published.

(A)

problems of the kind that Robert describes are


worse in Crownsbury than they are in other

The argument in Mikkeli's defense depends on

cities
Robert's proposal is intended to address a
problem that Robert does not in fact intend

(A)
(B)

Mikkeli has never met Halden

iC)

Norway
nobody related the plot of Halden's book in
detail to Mikkeli before Mikkeli wrote his

to patrol every street. So the city should install


speed bumps and signs warning of their presence
on residential streets to slow down traf8c.

Sheila: That is a bad idea.

People who are driving too

fast can easily lose control of their vehicles when


they hit a speed bump.

1.
BO

.=

(B)

tr

o
UI
o
o

(C)

E
I

(D)

IE

.g

bto

(E)

2,

it to

address
with speed bumps and warning signs in place,

there would still be drivers who would not


slow down to a safe speed
most of the people who are affected by the
problem Robert describes would be harmed
by the installation of speed bumps and
warning signs
problems of the kind that Robert desoibes do
not occut on any nonresidential streets in

the

'

assumption that

(D)
(E)

Halden's book did not become popular in

book
there is a common European myth to which
both authors referred subconsciously in the
books in question
Mikkeli is not familiar with Old lcelandic, an
extinct language related to an earlier form of

Norwegian

Crownsbury

Sources: PrepTast 37, Sectton


PrepTest 36, Sectlon

198

4, euestions 23
7, Questions 3

I(APLAN

PDF compression, OCR, web optimization using a watermarked evaluation copy of CVISION PDFCompressor

Assumption Questions

3.

Barnes: The two

ne\.vest employees at

this company

5.

Conservationist: The population ofa certain


wildflower is so small that the species is

have salaries that are too high for the simple


tasks normally assigned to new employees and
duties that are too complex for inexperienced

headed for extinction. However, this


wildflower can cross-pollinate with a closely
related domesticated daisy, producing viable
seeds. Such cross-pollination could result in a

workers. Hence, the salaries and the


complexity of the duties of these two newest
employees should be reduced.

significant population of wildflower-daisy


hybrids. The daisy should therefore be
introduced into the wildflower's range, since
although the hybrid would differ markedly
from the rvildflower, hybridization is the only
means of preventing total loss of the
wildflower in its range.

Which one of the following is an assumption on


which Barnes's argument depends?

(A)
(l.i;
((.1
(ti

The duties of the two newest employees are


not less complex than any others ia the
company.
it is because of the complex duties assigned
that the two newest employees are being paid
more than is usually paid to newly hired
employees.

'fhe fwo newest employees are not experienced


at their occupations.
Barnes was not hired at a higher-than-average
starting salary.
The salaries of the two newest employees are
no higher than the salaries that other

(E)

companies pay for workers with a similar


level of experience.

Which one of the following is an assumption on


which the conservationist's reasoning depends?
(A)

The wildflorver currently reproduces only by

(B)

The domesticated daisy was bred from wild


plants that once grew in the wildflower's

(c)

Increasing the population of the wildflower


rvill also expand its range.
Wildflower-daisy hybrids will be able tt'
reproduce.
The domesticated daisy will cross-pollinate
with any daisylike plant.

forming

range.

(D)
(E)

4,

Political opinion and analysis outside the


nrainstream rarely are found on television talk
shows, and it might be thought that this state of
affairs is a product ofthe political agenda ofthe
television stations themselves. In fact, television
stations are driven by the same economic forces as
sellers of more tangible goods. Because they must
attempt to capture the largest possible share of the
television audience for their shows, they air only
those shorvs that will appeal to large numbers of
people. As a resultlpolitical opinions and analyses
aired on television talk shows are typically bland and
innocrious,
T'he explanation offered by the author of the passage
makcs the assumption that

i.\
t.

j:i,

ii )
r,I--r

(fl)

television station executives usually lack a


political agenda of their own
'uland and innocuous political opinions and
analyses are generally in the mainstream
political analysts outside the mainstream are
relatively indifferent to the effect their
analyses have on television viewers
most television viervers are prepared to argue
against allowing the expression of political
ophions and analyses with which they

seeds.

Bacteria from food can survive for several days on the


surface of plastic cutting boards, but bacteria can
penetrate wooden cutting boards almost immediateln
leaving the surface free of contamination. Therefore,
wooden cutting boards, unlike plastic cutting boards,
need not be washed in order to prevent their
contaminating food that is cut on them; wiping them
off to remove food debris is sufficient.

Which one of the following is an assumption on


rvhich the argument depends?

(r\)
(B)
(C)
(D)
(E)

F
0
q!,

g!.

n
G
gl

|'l

o
=.
=
o(l

Washing plastic cutting boards does not


remove all bacteria from the surface.
Prevention of bacteria contamination is the
only respect in which wooden cutting boards
are superior to plastic cutting boards.
Food that is not already contaminated with
bacteria can be contaminated only by being
cut on contaminated cutting boards.
Bacteria that penetrate into wooden cutting
boards do not reemerge on the surface after
the cutting boards have been used.
Washing wooden cutting boards kills bacteria
below the surface of the cutting boards.

disagree

the political opinions of television station


executives are not often reflected in the
television shows their stations produce

Sources: PrepTest 29, Sectlon 7, Questlon 5


PrepTest 29, Section 4, Questlons 5 and 8
PrepTast 24, Sectlon 3, Questlon 5

E:9

lee

PDF compression, OCR, web optimization using a watermarked evaluation copy of CVISION PDFCompressor

ISAT Mostery Practice

7,

a method of producing text by means of


raised dots that can be read by touch. A recent
development in technology will allow flat conrputer
screens to be made of a material that can be heated
in patterns that replicate the patterns used in braille'
Since the thermal device will utilize the same symbol
system as braille, it follows that anyone who is
accustomed to reading braille can easily adapt to the

Braille is

9.

questions as, "FIow am I different from all other


creatures?" Therefore, philosophicai thought is
unique to humans.
The conclusion in the passage above relies on which
oire of the following assumptions?

use of this electronic system.

Which one of the following is an assumption on


rr'i:ich the conclusion depends?
, .i j
Braille is the only symbol system that can be
. readily adapted for use with the new thermal

.
'
bo

,=
E .

o
(,l
G

8.

E
I

I
b0
o
J

IT

'
,

'
,:

Itrrr:r:,1 with a financial crisis, Upland University's


board of trustees reduced the budget for the
unir,ersity's computer center from last year's $4
rnillion to $1.5 million for the coming year. However,
the center cannot operate on less than $2.5 million'
Silr,;e the board cannot divert funds from other
pr'.si.tms to the computer center' there is no \^ray
ti: rl rrri: center can be kept operating for the coming

i*,'

j r,r

.,

,
'

j.

, :.r

i'

i,rrclusion of the argument is properly drawn

if

l, one of the following is assumed?


'lhe computer center did not use all of the $4
million that was budgeted to it last year.
The budgets of other programs at the
university were also reduced.
fhe computer center has no source of funds
other than those budgeted to it for the coming
year by the university's board of trustees'
No funds from any Program at the university
can be diverted to other programs.
The board oftrustees at the universityvalue
other programs at the university more highly
than they do the computer center.

(A)
(E )

Human language is unique to humans.


Apes are incapable of thinking in human

i
(D)

Philosophical thought can be expressed only in


human language.
Speaking in human language is easier than
thinking in human language.
It is more difficult to learn human language
than to express phil<isophical questions.

(.--

screen,

Only people who currently use braille as their sole


medium for reading text will have the capacity to
adapt to the use of the thermal screen.
People with the tactile ability to discriminate
symbols in braille have an ability to discriminate
similar patterns on a flat heated surface'
Some symbol systems encode a piece of text by
using dots that replicate the shape of letters of
the alphabet.
Eventually it will be possible to train people to
read braille by first training them in the use
of the thermal screen.

Even though apes are the only nonhuman creatures


able to learn human language, no ape has ever used
its human language skill-q to ask such philosophical

(Lr
10.

Ianguage.

A nerv medication for migraine seems effective, but


there is concern that the medication might
exacerbate heart disease. If patients with heart
di-oease take the medication under careful medical
supervision, howeverrharmfrrl side effects can
definitely be avertedSilhe concern about those side
ellects is thus unfounded.
,

-i

he argument depends on which one of the

ii-' il 1;11,in g

,.i-)
,

ij

i
.

(I])
(E)

assumptions?

The new medication actually is effective when


taken by patients with heart disease.
No migraine sufferers with heart disease will
take the new medication except under careftrl
medical supervision.
Most migraine sufferers who have taken the
new medication in trials also had heart
disease.

The new medication has various other side


effects, but none as serious as that of
exacerbating heart disease.
The new medication will displace all migraine
medications currently being used.

Sources.'Preplest 22, Eectlon 2, Questlon


Preplest 22, 9ectlon 4, Questlons
PrcpTest

2!,

Sectlon

and

lJ.

2, Questlon 3

200
PDF compression, OCR, web optimization using a watermarked evaluation copy of CVISION PDFCompressor

Assumption

ll.

Copyright laws protect the rights of writers to profits


earned from their writings, whereas patent laws
protect inventors'rights to profits earned from their
inventions. In Jawade, when computer-software
writers demanded that their rights to profit be
protected, the courts determined that information
written for a machine does not fit into either the
copyright or the patent category. Clearly, therefore,
the profit rights of computer-software writers remain
unprotected in lawade.

13. Manager: Our new computer network,

the purpose
of which is to increase productivity, can be
installed during the day, which would disrupt
our employees'work, or else at night, which
' would entail much higher installation
charges.
Since saving money is important, we should
have the nefwork installed during me day.

The manager's argument assumes which one of the


foliorving?

Which one of the following is an assumption on


which the argument depends?

(A)

iB)
(C)
(D)
(E)

12.

Computer-software writers are not an


influential enough group in Jawade for the
government to consider modifring existing
copyright laws in order to protect this group's
profit rights.
No laws exist, other than copyright laws and
patent laws, that would protect the profit
rights of computer-software writers in

(A

i:i

equipment installed at night.

1{_)

{D)

One of the effects of lead poisoning is an


inflammation of the optic nerve, which causes those
who have it to see bright haloes around light sources.
In order to produce the striking yellow effects in his
"Sunflowers" paintings, Van Gogh used Naples
yellow, a pigment containing lead. Since in his later
paintings, Van Gogh painted bright haloes around
the stars and sun, it is likely that he was suffering
from lead poisoning caused by ingesting the

(E)
14.

(r\:
(B

((l)
in'i
(E)

In Van Gogh's later paintings he painted some


things as he saw them.
Van Gogh continued to use paints containing
lead after having painted the "Sunflowers"
paintings.
Van Gogh did not have symptoms of lead
poisoning aside from seeing bright haloes
around light sources.
The paints Van Gogh used in the "Sunflowers"
paintings had no toxic ingredients other than

during a daytime installation would be less


than the difference between daytime and
nighttime installation costs.
A daytime installation would be completed by
no larger a crew and would take the crew no
more time than would a nighttime installation.
Once the network has been installed, most of
the companyt employees will be able to use it
immediately to increase their productivity.
Most of the company's employees would be
able to work productively while a daytime
installation is in progress.

The symptoms of mental disorders are behavioral,


cognitive, or emotional problems. Some patients with
mental disorders can be effectively treated with
psychotherapy. But it is now known that in some
patients mental disorders result from chemical
imbalances affecting the brain. Thus these patients
can tre cffectively treated only with medication that
rviil redr.tce or correct the imbalance.
The argurnent depends on assuming which one of
the foliorving?

(A)

pigments he used.
Which one of the following is an assumption on
l.hich the argument relies?

The monetary value of the network equipment


rvould not exceed the cost of having the
The monetary value of any productivity lost

lawade.

Most of the computer software used in Jawade


is imported from other countries.
Computer software is more similar to writings
covered by copyright laws than it is to
inventions covered by patent laws.
Copyright laws and patent laws in Jawade have
not been modified since their original
adoption.

9rlg!:

it i
{

([,;

fr

E.
f.t

g
v
o
Ol

a
o
-

=.
oe

'lieatment by psychotherapy can produce no


effective reduction in or correction of
chemical imbalances that cause mental
disorders.
I r:aatment with medication always shows faster
rc'sults for patients with mental disorders
than does treatment with psychotherapy.
\lost mental disorders are not the result of
ehemical imbalances affecting the brain.
N{edication is always more effective in treating
patient.s with mental disorders tlan is
psychotherapy.
teatment with psychotherapy has no efl'ect on
mental disorders other than a reduction of
the symptoms.

lead.

The effects of Naples yellow could not have


been achieved using other pigments.

Soutces,' PrepTest
PrepTest

21, Section 2, Questlon 6


21, Sectlon 3, Qusstlon 73

PrepTest 24, Sectlon


PrcpTest 2O, Sectlon

!,

Questlon 3

4, Question

r(APrA9 2Ol
PDF compression, OCR, web optimization using a watermarked evaluation copy of CVISION PDFCompressor

ISAT Mastery Proctice


15.

John works five days each week except when on


vacation or during weeks in rvhich national holidays
occur. Four days a week he works in an insurance
company; on Fridays he works as a blacksmith. Last
week there were no holidays, and John was not on
vacation. Therefore, he must have rvorked in the
insurance company on Monday, Tuesday, Wedngsday,
antl Thursday last week.

t7.

lohn never takes


r.;eek in length.

'
)

i.ll
i

16.

.=

tr

o
o
o

I,!

&
I

(E

l'lany medical researchers do not agree to


serve as reviewers when their own research
in a critical phase.

1i.;

Reviewers for many medical journals are not


themselves medical researchers.
People rvould use nerv medical information
even if it were not first published in peer-

(E)

il

i rllol'ing is assumed?
critic enjoys writing about art works
rirat he or she dislikes intensely.
i ii art critics find it difficult to discover art
.,,'orks that truly satisfy them.
,'! ',r.ork of art that receives extensive critical
attention can thereby become more widely
. l'.noryn than it otherwise would have been.
fhe greatest works of art are never recognized
as -.uch until long after the time of their
:,.o atrt

creation.

I'he greatest works of art are works that


inevitably satisfu all critics.

iire argument depends?

(A)
'

(.

thl

Public health will improve more quickly in the wake


of new medical discoveries if medical researchers
abandon their practice of rvaiting until their findings
are published in peer-reviewed journals before
ilforming the press of important research results.
This is because the public release of new medical
information allows people to use that information in
order to improve their health, but the peer-review
process is unavoidably very slow.
u,,'1r:,"-lr

bo

l! )

nor on Sunday.
There were no days last week on which Iohn
both worked in the insurance company and
also worked as a blacksmith.

\{,Ihich one of the following is an assumption on

.g

:r,-'

i ,
: f-

vacation of more than one

Every day last week that John lvorked, he


rvorked for an entire rvorkday.
J ohn does not take vacations in weeks in which
national holidays occur.
Last week John worked neither on Saturday

(E)

bto

one

rii

\\/high one of the following is an assumption on


rlirich the argument depends?

(.,r.i

Many artists ciaim that art critics find it is easier to


write about art that they dislike than to write about
art that they like. Whether or not this hypothesis is
correct, most art criticism is devoted to art works
that fail to satisfy the critic. Hence it follows that
rnost art criticism is devoted to rvorks other than the
gieatest rvorks of art.
'l'ir,: r,r-.uclusion above is properly drawn if which

18.

Ed'rrcational television is a contradiction in terms.


trVLile a classroom encourages social interaction,
television encourages solitude. School is centered on
the development of language, but television depends
upon constantly changing visual images. And in a
cla:;sroom, fun is merely a means to an end, but on
teisi'ilion it is the end in itself.

llu,;rr rvirich one of the following assumptions does


::.',r.il;rx'rely in the passage?
(,i i i ire classroom should not be a place where

tlic

( 1l

is

revierved journals.
'Ihe peer-review process could be speeded up
enough to produce a significant improvement
in public health.
New medical information that is first
published in peer-revierved journals does not
usually receive public attention.

!.{

.i

{i:;
i

i)

,lnvone has fun.


)nly experiences that closely resemble what
takes place in the school environrfient can be
educational.
"l-elevision programs
reinforce some of the
values of the school environment.
E<iucational television programs are
qualitatively better than most other television
programs.
-fhe potential
t

of television as a powerful
learning tool has not yet been realized.

7j

Sources.' PrepTest 2O, Section 4, Quesilon


PrepTest !7, Section 2, Questlon t2
PrepTest 77, Sectlon 3, Questlon t4
PrepTest !O, Section
Questlon 7

t,

202

I(A

LAN

PDF compression, OCR, web optimization using a watermarked evaluation copy of CVISION PDFCompressor

Assumption Questions
rvhere there is
an alcohol problem, so unless something is
done about the alcohol problem at this

Sally: t cannot study at a university

19.

university,

I'll

2t. A favored theory to explain the extinction of


dinosaurs, together with many other species, has
been the globally catastrophic collision of a large
asteroid with the Earth. Supporti-ng evidence is an

have to transfer to a universin'

exlraterrestrial chemieal element in a layer of dust


found rvorldwide at a geological level laid down
conteilrporaneously with the supposed event. A nelv
conrpeting theory contends $at any asteroid itnpaet
was irrelevant, bec.ruse it rv#.nrassive volcirnic
activity that cansed the extinctions by putting
enough dust into the atmosphere to cool the planet )The l)eccan region of lndia contairrs extensive
volcalic flolvs that occurred within the same time
period as the supposed asteroid impact and the

where there are no fralernities.

Yolanda: I dont agree that fraternities are


responsible for the alcohol problem at this
university. Alcohol problems exist at all
univ-ersities, including those rvhere there are
no fraternities. We all should become more
aware of alcohol abuse. lt's not simply a
fraternity problem; it's a cuitural problem.

Wliiclr orrc of the following is an assumption on

n'lii;h rellv's argument

(Ar
( Ft)

'

,,,tinctions.

dePends?

1.irrr universities have fraternities.

hr

''-;trthing rvill- be done about the alcohol


l;r'oblem at Sally's university-.
((,, . t lcohol problems are becoming more
- *'idespread at universities,
{'i i r r lionre fraternity rnembers who drink alcohcl.
bc'verages are too young to do so legally-'
ii i - fhere could be univer.sities that have no

I'i ,rfi and-burn agriculture involves burling several


,iL:.ri of forest, leaving vegetable ash that provides
arr::rlc fslliliTer for three or four years of bountiful
- l *ils, On the' cleared land nutrients leach out of the
soii, horvever, and the land becomes too poor to
sr.r!,.port agriculture. Netv land is then cleared by
burning and the Process starts again' Since most
frrrming in the tropics uses this method, forests in
tlrr: rci;rr-rir rvill eventually be permanently
c

r.tdie itir"'tl.

'I-1rc

i\)

arq,rment depends on the assumption that

'(B

.lrriests in the tropics do not regenerate well


lnough to restore themselves once they have
been cleared by the slash-and-burn method
cone other methods of egriculture are not as

'

destructive to the environment in tropical


regions as the slash-and-burn method is
forests in the tropics are naturally deficient in
nutrients that are needed to support the
grorvth of plants that are not trative to those

i
(f, I

regions

llli
(Fi

slash-and-burn agriculture is particularly


suitable for farming in tropical areas
slash-and-burn agriculture produces a more
bountifirl crop than do other agriculture
.

methods for the iirst year

lJ,

(t

i-,

20,

.rre rr.

iJ

theory assumes that

ihe massive volcanic activity was not caused bv


the impact of an asteroid
,'<l individual dinosaurs survived the impact of
the asteroid. if it occurred
r he extinctions took place over a longer time
period than they would have if caused by the
inrpact of an asteroid

^
a?

other lolcanic eruptions rvere not occurring ;rt O


the same time as those in the Deccan region Oq
it is not possibleto determine which would havc
F'
occurred first, the volcanic flows in the Deccan Ot
region or the supposed impact of an asteroid n

2)..

()rrc of the requirements for admission to the


O
Lunnville Roller Skatrng Olub is a irisir degrce of skiU tt
in roller skrrting. f'he club president has expressed 9!
concern that th"e club rnayiave discriminaiecl against I
qualified women in its admissions this year. Yet half
of the applicants adrnitted to the club this year were =.
5
women. This proves that there was no discrimination OQ
against qualified women applicants in the club's
eclmissions this year.
\\rl i, h o ne of the following is an assumption on rvhich
il'ri' , rr1,'lusion of the argtrment depends?
r

\
i

i;rdy

{I

ii:

a few

applicants were lbund to be qualifiect

rrnd were admitted to the club this year.

(I:.)

'c more than half of all the roller skaters in


I unnville are ll.omen.
''lo
more than half of all the roller skaters in
Lunnville are men.
-[his
year no more than half of the applicants
who met all the qualifications for admission
to the cJub rvere \{'omel{.

'fhis year no more than half of the rnembers ot


the club's committee that rnakes decisions
about applicants' qualifications were men.

Sources: PrepTest 70, Sectlon !, Quastion 3


PrcpTest t0, Sectlon
Quesfions / and 15
PrapTest 7O, Seetion 4, Question 4

l.

KAPLA9
.___/

2O3

PDF compression, OCR, web optimization using a watermarked evaluation copy of CVISION PDFCompressor

ISAT Mostery

23.

Pradice

Advertisement: Attention pond owners!


Ninety-eight percent of mosquito larvae in a
pond die within minutes after the pond has
been treated with BTI. Yet BTI is not toxic to
fish, birds, animals, plants, or beneficial
insects. So by using BTI regularly to destroy
their larvae, you can greatly reduce
populations of pesky mosquitoes that hatch in
7'our pond, and you can do so without
diminishing the populations of fish, frogs, or
beneficial insects in and around the pond.
\A'hich one of the following is an assumption on
r.,,hich the argument depends?

(.\l

(j)

bo
II

i11)\'

tr
tr

o
o
Itr
o

E
I

tE 24.

,g

EO

'

'i

he most effective way to control the numbers


,rf mosquitoes in a given area is to destroy the
mosquito larvae in that area.
?opulations of mosquitoes are not dependent
on a single body of water within an area as a
place for their larvae to hatch and develop.
'lhere are no insect pests besides mosquitoes

that pond owners might want to eliminate


from in and around their ponds.
-llhe effectiveness of BTI in destroying
mosquito larvae in a pbnd does not require
the pond owner's strict adherence to specific
application procedures.
The fish, frogs, and beneficial insects in and
around a pond-owner's pond do not depend
on mosquito larvae as an important source of
food.

25,

The economies of some industrialized countries face


the prospect of large labor shortages in the decades
ahead. Meanwhile, these countries will have a vast
number of experienced and productive older workers
who, as things stand, will be driven from the work
force upon reaching the age of sixty-five by the
widespread practice of requiring workers to retire at
that age. Therefore, if the discriminatory practice of
mandatory'retirement at age sixty-five were
eliminated, the labor shortages facing theseeconorlrii.r rvould be averted.

l hr. :it-grntent assumes

{A)
ili

,ilr

(1

that

ulLlci workers have acquired skills that are


' .i I remely valuable and that their younger
,'tileagues lack
'.:

i)ikers in industrialized countries are often

,,nprepared to face the economic


' ('nsequences of enforced idleness
j,":ge number of workers in some
'
i i r<lustrialized countries would continue
x'orking beyond the age of sixty-five if workers
in those countries were allowed to do so
rrandatory retirement at age sixty-five lyas
first instituted when life expectanry was
considerably lower than it is today
a substantial proportion of the population of
officially retired rvorkers is actually engaged

in gainful employment

English and the Austronesian language Mbarbaram


both use the word "dog" for canines. These two
languages are unrelated, and since speakers of the
two languages only came in contact with one another
long after the word "dog" lvas first used in this way in
either language, neither language could have
borrowed the word from the other. Thus this case
shows that sometimes when languages share words
that are similar in sound and meaning the similarity
is due neither to language relatedness nor to
borrorving.
The argument requires that which one of the
follo..ving be assumed?
' r.;

,
:n

ii ' ,
,

ii-i) '
(t

.
,

English and Mbarbaram share no words other


than "dog."
Several languages besides English and
Mbarbaram use "dog" as the word for canines.
l.lsually when two languages share a word,
ihose languages are related to each other.
l here is no third language from which both
English and Mbarbaram borrowed the word

"dog"'

If two unrelated languages share a word,


speakers of those trvo languages must have

come in contact with one another at some time.

Sources: PrepTest tO, Sectlon 4, Questlon 18


Preplest 9, Section 2, Questlon 27
PrepTest 9, Sectlon 4, Questlon 6

204
PDF compression, OCR, web optimization using a watermarked evaluation copy of CVISION PDFCompressor

Assumption Questions
26.

Myrna:

People should follow diets in which fat


represents no more than 30 percent of total
calories, not the 37 percent the average diet in
this country contains.

28. Physiological

research has uncovered disturbing


evidence linking a number of structural disorders to
jogging. Among the ailments seemingly connected

with this now-popular sport are spinal disk


displaceme4ts, stress fractures of the feet and ankles,
knee and hip joint deterioration, and tendonitis.
Furthermore, these injuries do not occur exclusively
among beginning runners-veteran joggers suffer an
equal percentage of injuries. What the accumulating
data suggest is that the human anatomy is not able to
withstand the stresses of jogging.

Rolandr If everyone in the country followed your


recommendation during his or her entire life,
just 0.2 percent would lenghen their lives at all'
and then only by an average of 3 months.
Modifying our diet is not worthwhile. A lifetime
of sacrifice spent eating an unappealing low-fat
diet is too high a price to pay for the chance of
extending that sacrifice for 3 months.

N,[vrna: But for everyone who dies early from

'".'irich one of the following is an assumption of the


igumr:'ri?

high-fat diet, many more people suffer from


serious chronic diseases because they followed
such diets.

. l;

Some accountants calculate with simple

27.

ri desirtble to live in such a way as to


. icngthen life as much as possible
.r lrrrv-fat diet cannot readily be made
appealing and satisfying to a person who
lollows it regularly
dict is the only relevant factor to consider in
computing influences on length of life
the difference in tastiness between a diet in
rvhich fat represents 30 percent of total
calories and one in which it represents 37
percent is not noticeable
not everyone in the country eats the average
diet
is

adding

"

inachines, and some use complex computers. One


can perform more calculations in less time with a
computer than with an adding machine. Therefore,
assuming the costs of using the two types of
machines are equal, an accountant who uses a
computer generally can earn more per hour than an
?lccountant who uses an adding machine,

l\4rich one of the following is an assumption that


lr.oukl make the conclusion in the passage a logical
-

ctt.

."

iiJ)

The jogger's level of experience is a factor


determining the likelihood of a jogging injury.
Some sports are safer for the human body than

il-l

I'he human species is not very durable.

jogging.

i;ntil

he rvas dismissed amid great controversy,


Hastings was considered one of the greatest
intelligence agents of all time. It is clear that if his
dismissal was justified, then Hastings was either

incompetent or else disloyal. Soon after the dismissal,


however, it was shown that he had never been
incornpetent. Thus, one is forced to conclude that
Hastings must have been disloyal.

:
.r,
'I

i!))
(E)

q3.

f!

g
n

o
g,

t'l

\\rh ir.lr one of the following states an assumption


rr:.,.:r r,'hich the argument depends?
i

Hastings's dismissal was justified.


Hastings was a high-ranking intelligence
officer.
The dismissal of anyone who was disloyal
rvould be justified.
Anyone whose dismissal was justified was
disloyal.
If someone was disloyal or incompetent, then
his dismissal was iustified.

o
5
-l

q(I
=

'
i :.,
i.4

r' )

, I
"

iie lirrk between jogging and certain structural


disorders appears to be a causal one.
Iogging causes more serious disorders than
other sports.
.

;;:,. ,iid'-s iirgumrnt assumes that

'

lr)

More accountants use computers than use


adding machines.
The more hours an accountant spends on the
job, the more money he or she will earn.
The more calculations an accountant performs,
the more money he or she will earn.
An accountant rvho uses an adding machine
can charge a higher hourly rate than one who
uses a computer.
In general, accountants vary in terms of the
number of calculations they make and the
amount of monev thev earn.
Sources,' PrepTest

9, Sectlon 4, Questlons 70 and 72

Preplest 6, Sectlon 2, Questlon 2


Preplest 6, Secfion 3, Questlon 9

205

PDF compression, OCR, web optimization using a watermarked evaluation copy of CVISION PDFCompressor

Mastery Practice

LSAT

30

'llhe seventeenth-century physicist Sir Isaac Newton


is remembered chiefly for his treatises on motion and
gravity. But Newton also conducted experiments
secretly for many years based on the arcane theories
of alchemy, trying unsuccessfully to transmute
common metals into gold and produce rejuvenating
elixirs. If the alchemists of the seventeenth century
had published the results of their experiments,
chemistry in the eighteenth century would have been
more advanced than it actually was'

32.

lArhich nne of the following assumptions would allow

excellence.

the coqclusion concerning eighteenth-century


chemistry to be properly drawn?

(A)

ill)
bo
s
II

tDl

-a
-0

{B)
-

{a

|\'
c,
&,

-IE
I

trI

bo

lt.

Annnals with a certain behavioral disorder have


unusually high levels of aluminum in their brain
tissue. Since a silicon-based compound binds to

aluminum and prevents it from affecting the brain


tissue, animals can be cured of the disorder by being
treated rvith the comPound.
Tl're argument is based on which one of the following
ascr
{

rnptions?

r\'}

r
r1 ')

Which one of the following is an assumption on


rvhich Senator Beton's argument is based?

Scientific progress is retarded by the reluctance


ot historians to acknowledge the failures of
sorne of the great scientists'
i\dvances in science are'hastened when reports
of experiments, whether successful or not, are
available for review by other scientists.
l'ien'ton's work on motion and gravity would
nLlt have gained wide acceptance if the results
of his work in alchemy had also been made
nublic.
Iircreasing specialization-within the sciences
nrakes it difficult for scientists in one field to
understand the principies of other fields.
'I'he seventeenth-century alchemists could have
achieved their goals only if their experiments
had been subjected to public scrutiny.

lB)

ii

1)

{Il

Animals with the disorder have unusually high


but invariable levels of aluminum in their

l:rain tissue.
rninum is the cause of the disorder rather
tiran merely an effect of it.
tniroducing the compound into the brain

Ai

tr

ii.csue has no side effects.

['hc amount of the compound needed to


nr'rltralize the aluminum in an animal's brain
tissrre varies depending upon the species.
i\lnrninum is never present in normal brain
iissue,

The stated goal of the government's funding program


for the arts is to encourage the creation ofworks of
artistic excellence. Senator Beton claim3, however,
that a government-funded artwork can never reflect
the independent artistic conscience of the artist
because artists, like anyone else who accepts financial
support, will inevitably try to please those who
control the distribution of that support. Senator
Beton concludes that go\ernment funding of the afts
not only is a burden on taxpayers but also cannot
lead to the creation of works of true artistic

(AJ l;lost taxpayers have little or no interest in the


. creation ofworks oftrue artistic excellence.
(it) Goyrnmot funding of the arts is more
generous than other financial support most

artists receive.

({''
(l )

( fr

ifistribution of government funds for the arts


is based oir a broad agreement as to what
constitutes artistic excellence.

Once an artist has produced works of true


artistic excellence, he or she will never accept

government funding.
A contemporary work of art that does not
reflect the independent artistic conscience of
the artist cannot be a work of true artistic
excellence.

33.

School superintendent It is a sad fact that, until now,


entry into the academically best high school in our
district has been restricted to the children of people
who were wealthy enough to pay the high tuition.
Parents who were previously denied the option of
sending their children to this school now have this
option, since I am replacing the tuition requirement
with a requirement that allows only those who live in
the neighborhood of the school to attend.
The superintendent's claim about the effect of
repl.rcing the tuition requirement relies.on the

assumption that

(A)
: ir,

the residents of the school's neighborhood


tend to be wealthy
people other than those wealthy enough to

tuition are able to live in the


neighborhood of the school
people less wealthy than those who were able
to pay the old tuition are in the majority in
the district
there are no high schools in the district other
than the one referred to by the superintendent '
there are many people not wealthy enough to
have paid the old tuition who wish to have
their children attend the school
have paid the old

(D)
(E)

t,

Question 76
PrepTest 5, Sectlon 3, Questions 5,7, and 14

Sources.'Preprest 5, Section

206

I(APLAN

PDF compression, OCR, web optimization using a watermarked evaluation copy of CVISION PDFCompressor

Assumption Questions
34.

Although consciousttess seerns tc rrise fi'orn physical


processes' physical tireories c;lr', r'1r)1;iiI! on[y why
phi.sical systems ltirve ieit;ri,r "1r,,'ii,,tl :;tmctures and

36. Editorialist:

To ensure justice in the legal system,


citizens must be capable of criticizing anyone
involved in determining the punishment of
criminals. But when the legal system's purpose is
seen as deterrence, the system falls into the hands
of experto whose specialty is to assess how
potential lawbreakers are affected by the system's
punishments, Because most citizens lack
knorvledge about such matters, iustice is not then
ensulctl in the legal system,

horu these systelns pcrfornr \ r ,(,iis pirysical


fune rions. Thus, uo stricf 1,. r-'i:r's1.',il theory can
e-r1; l;r

consciotrstress.

l-, :onclusion of thl .,;':rrilite rrt follorvs


: ji one sf *ls fo|!ir,. ing is assumed?
.

r,

I t.r

logically

if

Physical theories can explain only physical


phenomena.

,
i

rl

ii:i

fhe editorialist's argument requires assuming which

An explanation of consciousness must


encompass more than an explanation of
physical structures and functions.
The physical structures and functions of
consciousness are currently unknown.
Consciousness arises from processes that are
entirely explainable by physical theories.
An explanation ofphysical structures atrd
functions must be formulated in strictly

one of the following?

{.4,i

physical terms.

35.

fr{)
V

Cultural historian: Universal acceptance of scientific


theories that regard human beings only as natural
obiects subiect to natural forces outside the
individualt control will inevitably lead to a
general decline in morality. After all, if people do
not believe that they are responsible for their
actions, they rvill feel unashamed when they act
iliinorallv, and a widespread failure of individuals
- il t'eel ashamed of theiiimmoral actions is bound
r,, lead to a general moral decline.
i irr'.or,:lusion drawn by the cultural historian
iriliir',','i icgi,:ally if rvhich one of the following is
.: i - 'r :lt(l {

:'1 .

i)

(1:

l
)
i

ii,man beings who regard themselves only

,.:

'
'

37.

The retail price of decaffeinated coffee is considerably


higher than that of regular coffee. However, the process
by which coffee beans are decaffeinated is fairly simple
and not very co-stiy. Therefore, the price difference
cannot be accor-rnted for by the greater cost of
providing dei:affeinated coffee to the consumer.
'l'he argumcnt relies on assuming which one of the

follor,

rn!:?

)
(l't)

ij{ ()LL'ssing regular coffee costs more


1, r'ot:essing decaffeinated coffee.

Srrrrrce does not enable human beings to


f r)Jrtrol natural forces,

as

latural objects rvill as a result lose their sense


o1' responsibility for their actions.
i'rople who have a sense of shame for their
;noral transgressions will feel responsible for
iheir actions.
5ome scientific theories hold that human
beings are not responbible for their actions.
Scientific explanations that regard human
beings as in some respects independent of the
laws of nature will not lead to a general
decline in morality.

Most citizens view justice as primarily


concerned with the assignment of
rrunishment to those who deserve it.
Irr order to be just, a legal system must
consider the effect that punishment will have
t'n individual criminals.
I he primary concern in a legal system is to
,:dminister punishments that are just.
1n a legal system, a concern for punishment is
incompatible with an emphasis on deterrence.
{iitizens without knowledge about how the
iegal system's punishments affect potential
lawbreakers are incapable of criticizing
experts in that area,

.\

i,

,.

, '

ol
I

o
o
o
o
I
J
lll

:'

than

i;;'jcl differences between products can


as

.;,Juction costs,
i , r ,r'1 i5
;

F
o
q!.

oe

srrrerally be accounted foi by such factors


-:";ply and demand, not by differences in
i

fr'

:,
'

little competition among companies

r {)focss decaffeinated coffee.

,,;ii coffee-sellers do not expect that


'!,sumers are content to pay more for
,rcaffeinated coffee than for regular coffee,
i:e beans used for producing decaffeinated
, offee do not cost much more before
processing than the beans used for producing
regular coffee.

2,
4,
PrepTest 36, Section 7,
PrepTest 36, Section 3,

Sources: PrepTest 37, Section


PrcpTest 37, Sectlon

Questton 5
Questlon 9
Questlon 20
Questlon 22

207

PDF compression, OCR, web optimization using a watermarked evaluation copy of CVISION PDFCompressor

Mostery Praaice

LSAT

38

Novelists cannot become great as long as they'remarn


in academia. Powers of observation and analysis, which
schools successfully hone,.are useful to the novelist, but
an inruitive grasp of the emotions of everyday life can
be obtained only by the kind of immersion in everyday
life that is precluded by being an academic.
.'rrl1irfu one of the following is an assumption on
, ',lrh the argument depends?

'.

.)

{l:)

XT

)^(
lo

.H@

E@
|n

fo{arian Anderson,.the famous contralto, did not take


success for granted. We know this because Anderson
had to struggle early in life, and anyone who has tEr
stmggle early in lit'e is ahle to keep a good
r:r:rspective on the wcr'id.
'I

rE

..'.

hc conclusion of the arllment follows logically


, r ich one of the follorving is assumed?

'i
: r:
''

&,
f,

Novelists require some impartiality to get an


intuitive grasp of the emotions of everyday life.
No great novelist lacks porvers of observation
and iinalysis.
Participation in life, interspersed with impartial
observation of life' makes novelists great.
Noveiists cannot be great without an intuitive
grasp of the emotions of everyday life.
Knowledge of the emotions of everyday life
cannot be acquired by merely observing and
analyzing life'

|tr

IJ
aa

rI

'

:!

40.

Political opinion and analysis outside the


mainstream rarely are found on television talk
shows, and it might be thought that this state of
affairs is a product of the political agenda of the
television stations themselves. In fact. television
stations are driven by the same economic forces as
sellers of more tangible goods. Because they must
attempt to capture the largest possible share of the
television audience for their shows, they air only
those shows that will appeal to large numbers of
people. As a result, political opinions and analyses
aired on television talk shows are typically bland and
innocuous.

An assumption made in the explanation offered by


the author of the passage is that

iA)
('tj

i,

''

if

r\lyone who succeeds takes success for granted.


.\nyone who is able to keep a good perspective
on the world does not take success for granted.
Anyone who is able to keep a good perspective
on the world has to struggle early in life.
,.\nyone who does not take success for granted
has to struggle early in life.
Anyone who does not take success for granted is
able to keep a good perspective on the world.

rnost television viewers cannot agree on which


elements of a particular opinion or analysis
arc most disturbing
ttrere are television viewers who might refuse
to watch television talk shows that they knew
w'ould be controversial and disturbing
< ach television viewer holds some opinion that
is outside the political mainstream, but those
opinions are not the same for everyone
ihere are television shows on which'economic
forces have an even greater impact than they
do on television talk shows
the television talk shows of different stations
resemble one another in most respects

4t. i.,ommercial

passenger airplanes can be equipped with


collision-avoidance radar system that provides pilots
rvith information about the proximity of other
airplanes. Because the system warns pilots to take
evasive action when it indicates a possible collision,
passengers are safer on airplanes equipped with the . .
system than on comparable airplanes not so equipped,
even though the system frequently warns pilots to
evade phantom airplanes.
a

Which one of the following is an assumption on


rvhich the argument depends?

(A)

Passengers feel no saf*:

with the radar


{i-j

i,

.,

D)

if,)
Sources: PrcpTest 35, Sectlon 7, Question 74
PrepTest 35, Sectron 4' questlon t4
Prcprest 29, Sectlon 4, Question 4
Prep.Test 26, Sectlon 2, Questlon 70

sysit:t'r.

ln airplanes equipped
than on comparable

airplanes not so :,'r; rripped.


Warnings given by a collision-avoidance
system about phantom airplanes are not
caused by distorted radar signals.
The frequency of invalid warnings will not
cause pilots routinely to disregard the
system's warnings.

Commercial passenger airplanes are not the


only planes that can be equipped with a
collision-avoidance system
The greatest safety risk for passengers traveling
on commercial passenger airplanes is that of a
midair collisios"

208

PDF compression, OCR, web optimization using a watermarked evaluation copy of CVISION PDFCompressor

42.

@
@

The Board of Trustees of the Federici Art Museum


has decided to sell some works from its collection in
order to raise the fulds necessary to refurbish its
galleries. Although this may seem like a drastic
remedy, the curator has long maintained that among
thb paintings that the late Ms. Federici collected for
the museum were several unsuccessful immature
works by Renoir and C6zanne that should be sold
becarrse they are of inferior quality and so add
nothing to the overall quality of the museum's
eoji.ir:tion. Hence, the board's action will not detract
ficr:i :he quality of the museum's collection.-

'flr*: ,..r:rclusion draulp depends on which one of the


foll

c',,,'':r

(1;
|

!l'i

.i

'

43.

A certain credit-card company awards its customers


bonus points for using its credit card. Customers can
use accumulated points in the purchase of brand
name merchandise by mail at prices lower than the
manufacturers'suggested retail prices. At any given
time, therefore, customers who purchase
merchandise using the bonus points spend less than
they rvould spend if they purchased the same
merehandise in retail stores.

l{&ic}r one of the following is an assumption on


',rlt,,.ir ihe argument depends?

(,t.,

rg assumptioils?

,'\rt speculators are unable to distinguish an


inferior painting by Renoir from a
,nasterpiece bY him.
1l of the paintings that the board of trustees
;ells will be among those that the curator
lecommends selling.
all of the paintings by Renior and Czanne
that are olvned by the Federici Art Museum
were purchased by Ms. Federici herself.
Only an avid collector of paintings by C6zanne
would be rvilling to Pay a high price for early
works by this artist.
A great rvork of art can be truly appreciated
only if it is displayed in a carefully designed
and well-maintained gallerY.

ili

i ,1

;:

fhe merchandise that can be ordered by mail


using the bonus points is not offered at lower
prices by other credit-card companies that
award bonus points.
The bonus points cannot be used by the creditcard customers in the purchase of brand
name rnerchandise that is not available for
purchase in retail stones.
The credit-card company does not require its
customers to accumulate a large number of
tronus points before becoming eligible to
order merchandise at prices lower than the
manufacturers' suggested retail price.
I'he amount credit-card customers pay for
shipping the merchandise ordered by mail
does not increase the amount customers
spend to an amount greater than they rvould
spend if they purchased the same
merchandise in retail stores.
fhe merchandise available to the company's
credit-card customers using the bonus points
is frequently sold in retail stores at prices that
are higher than the manufacturers'suggested
retail prices.

*
*

q3.

f'l

g
n
o
Ol

t'l'

0
=
3
II

oq

Sources; PrcpTest 25, Sectlon


PrepTest 20, Sectlon

4, Questlon 9
7, Questlon 77

209

PDF compression, OCR, web optimization using a watermarked evaluation copy of CVISION PDFCompressor

44.

plants purple coneflower and goldenseal. A cold


sufferer, skeptical of the claim that the mixture is an
effective cold remedy, argued, "Suppose that the
mixture u'ere an effective cold remedy. Since most
people with colds wish to recover quickly' it follows
that almost everybody with a cold would be using it.
Therefore, since there are many people who have
colds but do not use the mixture, it is obviously not

effective." '

lVhich one of the following is an assumption on

Each of the following is an assumption required by

rvhich the argument depends?

the skeptical cold sufferer's argument EXCEPT:

{r\)

mixture as a remedy for the common cold. The


mixture contains, among other things, extracts of the

(Ai

tk

(B)
(C)

bo
II

(t))

(E)

o
o

45.

Private industryis trying to attract skilled research


scientists by offering them high salaries. As a result,
most research scientists employed in private industry
now earn 50 percent more than do comparably
skilled research scientists employed by the government.
So, unless government-employed research scientists
are motivated more by a sense of public duty than by
their own interests, the government is likely to lose
its most skilled research scientists to private industry,
since none of these scientists would have problems
fi nding private-sector jobs,

Some people have been promoting a new herbal

Enough of the mixture is produced to provide


the required doses to almost everybody with a
cold.
The mixture does not have side effects severe
enough to make many people who have colds
avoid using it'
The mixture is powerful enough to prvent
almost everybodY who uses it from
contracting any further colds.
The mixture is widely enough known that
almost everybody with a cold is aware of it.
There are no effective cold remedies available
that many people who have colds prefer to the

lB)
',{,)
iD)

mixture.

IU

cl

(f,)

&
-IE
tJ

Government research scientists are less likely


to receive acknowledgment for their research
contributions than are research scientists in
the private sector.
None of the research scientists currently
employed by the government earns more than
the highest-paid researchers employed in the
private sector.
The government does not employ as many
research scientists who are highly skilled as
does any large company in the private sector
which employs research scientists.
The government does not provide its research
scientists with unusually good working
conditions or fringe benefits that more than
compensate for the lower salaries they receive.
Research scientists employed in the private
sector generally work longer hours than do
researchers employed by the government.

II

blD

0
J

Sources.'Preplest 20, Sectlon 7, Questlon 20


Prepfest 2O, Sectlon 4, Questlon 76

210

I(APtAN

PDF compression, OCR, web optimization using a watermarked evaluation copy of CVISION PDFCompressor

Assumption Questions
46.

The current move to patent comPuter programs is a


move in the wrong direction and should be stopped.
The patent system was originally desigrred solely to
protect small-time inventors from exploitation, not to
give large corporations control over a methodology.
Any computer program is merely the implementation
of a methodology.

48.

Mary

lnez:

\'\&ich one of the following is an assumption on


rvhich the argument depends?

(-'\j
lF

r(l)
(q)
(E)
A1

arisen.
Large corporations should not hold patents for
implementations of methodologies.
Small-time inventors who support the move to
patent comPuter programs act contrary to
their own best interests.

(E)

replication experiments assumes that

(C)
(Ul
(Fl)

There are many examples in history of


countries that were strong but used their
strength to commit atrocities. We should judge
a country by the morality of its actions, not by
its strength. If the actions are morally good,
the country is admirable.

\1*rich one of the following is a presupposition that


underlies Inez' argument?

(r\)
(B)
(C)
(D)

the original experiments had not been


described in sufficient detail to make an exact
replication possitrle
the fact that the originally reported results
'aroused controyersy made it highly likely that
they were in error
the theoretical principles called into question
by the originally reported results were
themselves based on weak evidence
the replication experiments were not so likely
as the original experiments to be marred by
faulty measurements
the researchers who originally reported the
controversial results had themselves observed
those results only once

At least one country is admirable.


Countries cannot be both strong and moral.
It is possible to assign moral weight to the
actions of countries.
The citizens of any country believe that
whatever their country does is good.
Countries should impose their standards of
morality on other countries by whatever

*
*

means nece$sary.

49.

Thomson, the discoverer of the electron and a


recipient of the Nobel Prize in physics, trained many
physicists, among them seven Nobel Prize winners, 32
fellows of the Royal Society of London,pnd 83
professors of physics. This shows thaqftre skills peeded
ior creative reiearch can be taught ancl)earned.)

J. J.

(A)
(B)
i{l)
{D)
(E)

l-

o(l

n'
!t
r-r

7
o

Which one of the following is an assumption on


lvhich the argument depends?

The argument of the scientists lvho conducted the

(B)

Our country should, above all, be


strong. Strength gains the respect of other
countries and makes a country admirable.

Computer programs should be developed not


only by large corporations but by small-time
inventors as well.
Implementing a methodology always requires
less creative effort than does true invention.
The issue of whether or not to patent
computer programs Presents the patent
system with problems that have never before

Scientists attempting to replicate certain


controversial results reported by a group of
experienced researchers failed to get the same results
as those reported. The conclusion drawn from this by
the scientists who conducted the replication
experiments was that the originally reported results
had been due to faulty measurements.

(A)

Ann:

Thomson was an internationally known


physicist, and scientists came from all over
the world to work with him.
All the scientists trained by I. I. Thomson were
renowned for their creative scientific r'esearch.
At least one of the eminent scientists trained
by I. I. Thomson was not a creative researcher
before coming to study with him.

J. J.

gl

n
o
3
-r
3

otl

Creative research in physics requires research


habits not necessary for creative research in
other fields.
Scientists who go on to be the most successfirl
researchers often receive their scientific

education in classes taught by renowned


research scientists.

Eources: PrepTeet 77, Sectlon 3, Questlon 3


fuepTest 9, Sectlon 4, Questlon 25
PrepTest 6, Sectlon 2, Questlons and

t5

I(APtAN

211

PDF compression, OCR, web optimization using a watermarked evaluation copy of CVISION PDFCompressor

50. Fire ants from Brazil now infest the soitl{ern

United

51. Head injury

is the most serious type of injury


sustained in motorcycle accidents. The average cost
to taxpayers for medical care for nonhelmeted
motorcycle-accident victims is twice that for their
helmered counterparts. )urisdictions that have
enacted rnotorcycle-helmet laws have reduced the
incidence and severity of accident-related head
injuries, thereby reducing the cost to taxpayers.
Therefore, to achieve similar cost reductions, other
jurisdictions should enact motorcycle-helmet laws.
For the same reason jurisdictions should also require
helmets for horseback riders, since horseback-riding
accidents are even more likely to cause serious head
injury than motorcycle accidents are.

States. Unlike queen fire ants in Brazil, two queens in


the United States share a nest. Ants froni these nests
are more aggressive than those from single-queen
nests. By destroying virtually all insects in the nest
area, these aggressive ants gain sole access to food
sourcs, and the ant population skyrockets. Since

certain predator insects in Brazil limit the fue-ant


population there, importing such predator insects
into the United States would be of overall benefit to
the environment by stopping the increase of the
fire-ant population in the United States.
Each of the following is an assumption made in the

argument EXCEPT:

(A)

*
*

(*)
'
'

bo

tr

tI

o
o
o
o

{{:l
(D)
'
(E)

lrihich one of the following is an assumption upon

The imported insects would not prove more


damaging to the environment in the United
States than are the fire ants themselves.
The predator insects from Brazil could survive
in the ecological environment found in the

,,vlr;ch the author's conclusion concerning helmets


iir', seback riders depends?

United States.
The especially aggressive fire ants from the
two-queen nests would not be able to destroy
the Brazilian predator insects.
The predator insects would stop the increase of
the ant population before the ants spread to

.'

'

i:

'

Medical care for victims of horseback-riding


accidents is a financial drain on tax funds.
The higher rate of serious head injury suffered
by victims of horseback-riding accidents is
due to the difference in size betrveen horses

i.

states that are farther north.

:)

The rate ofincrease ofthe fire-ant population


would not exceed the rate at which the
predator insects could kill the ants.

i;.)

for

and motorcycles.
The medical costs associated with treating
head injuries are higher than those for other
types of injury.

Most fatalities resulting from horseback-riding


and motorcycle accidents could have been
prevented if the victims had been wearing
helmets.

When deciding whether to enact helmet laws


for motorcyclists and horseback riders, the

jurisdiction's primary concern is the safety of

.g

its citizens.

bo

52.

Twenty years ago the Republic of Rosinia produced


nearly 100 million tons of potatoes, but last year the
harvest barely reached 60 million tons. Agricultural
researchers, who have failed to develop new
higher-yielding strains of potatoes, are to blame for
this dccrease, since they have been concerned only with
thcir own research and not with the needs of Rosinia.

\\iiir; h one of the following is an assumption on


rvlri, lr the argument depends?
{,ri

.j;

'

Ij

i',

Any current attempts by agricultural


researchers to develop higher-yielding potato
strains are futile,
Strains of potatoes most commonly grown in
Rosinia could not have produced the yields
last year that they once did.
Agricultural researchers often find concrete
solutions to practical problems when
investigating seemingly unrelated questions.
fluctuations in the size of the potato crop
";\iide
over a twenty-year period are not unusual.
Agricultural research in Rosinia is funded by
goYernment grants.

Source: Prepfest 6, Sectlon

3, Questlons 3, 7, antl 75

212 SetA'9
PDF compression, OCR, web optimization using a watermarked evaluation copy of CVISION PDFCompressor

Assumptian Questions

t
53.

Tbe reforms to improve the qualityof public


education thet have been initiated on the part of
suppliers of public education have been insufficient.
Therefori,4pforms must be demanded by consumers.
Parents should be given government vouc.hers with
which to payfor their drildren's education and
should be allowed to choose the schools at which the
vouchers will be spent. To attract studenG,
academically underachieving schools will be forced

to improve their acadernic offerings.

,,

The argument assumes that

(A)
(B)

(c)
(D)
(E)

in selecting schools parents would tend to


prefer a reasonable level of ecadenic qudity
to greatr sports oPportunities or more
convenient location
imprormnentin the acadenric ofieringp of schools
willbe enforcdbythe discipline of the job
market in which graduating students aompete
there is a single best way to educate students
children are able to recognize whic.h sctrools
are better and would influence their parents'
decisions
schools would each improve all of their

academic offeringp and wouldnot tend to


specialize in one particular field to the
exclusion of others

9ource; Prcptatt E, Sectlorr S, Queg,tlon

t0

{l[9

213

PDF compression, OCR, web optimization using a watermarked evaluation copy of CVISION PDFCompressor

|SAT Mastery Practice


54.

critic The meaning of a literary work is not


fixed but fluid, and therefore a number of equally
valid interpretations of it may be offered.

Literary

Interpretations primarily involve imposing


meaning.ora-literary work rather than
discovering meaning in it, so interpretations need
not consider the writer's intentions, Thus' irny
interpretation of a literary work tells more about
the critic than about the writer.

56.

Editorial: The threat of harsh punishment for

\.{*'liich one of the following is an assumption

reiiuired by the literary critic's argument?

i.'

There are no criteria by which to distinguish


the validity of different interpretations

(li)

>(

i(lr

*
*

(Ii

()'I
u0
II

tr
a-

Vague laws set vague limits on people's freedom,


*'hich makes it impossible for them to know fgr
certain whether their actions are legal, Thus,.irnder
vague laws people cannot feel secure. ,

55.

o
UI
G
o
-l

'i'he conclusion follows logically if which one of the


i,',ilowing is assumed?

&

-tg
lr,
tI

bo

r
(Bl
(.4

of

literary works.
A meaning imposed on a literary work reflects
facts about the interpreter.
A writer's intentions are relevant to a valid
interpretation of the writert work.
The true intentions of the writer of a work of
literature can never be known to a critic of
that work.
The deepest understanding of a literary work
requires that one know the writer's history'

if they know for


certain whether their actions are legal.
If people do not know for certain whether
their actions are legal, then they might not
People can feel secure only

feel secure.

t
(D)

(C

(E)

If people know for certain whether their


actions are legal, they can feel secure.
People can feel secure ifthey are governed by
laws that are not vague.
Only people who feel secure can know for
certain whether their actions are legal.

transgression usually decreases one's tendency


to feel guilt or shame for committing that
transgression, and the tendency to feel guilt or
shame for committing a transgression reduces
a person's tendency to commit transgressions,
Thus, increasing the severity ofthe legal
penalties for transgressions may amplify
people's tendenry to ignore the welfare of
others.

Y/hich one of the following is an assumption


required by the editorial's argument?

(A)
(ij

{(,)
(D)
(E)

Legal penalties do not determine the morality


of an action.
At least some actions that involve ignoring the
welfare of others are transgressions.
People who are concerned about threats to

their own well-being tend to be less


concerned about the welfare of others.
The threat of harsh punishment deters people
from committing transgressions only if this
threat is at least sometimes carried out.
Everyone has at least some tendency to feel
guilt or shame for committing extrernely
severe transgressions.

We learn to use most of the machines in our lives


through written instructions, without knor':ledge of
the machines'inner workings, because mosl
machines are specifically designed for use by
nonexperts. So, in general, attaining technoit'gical
expertise would prepare stude rrts for ttimorlo'rv's job
market no better than rvould a more traditiunal
education stressing verbal and quantitative skills.
Thr, rrgument depenils on assurning rvhich one of
thc r'ollowing?

(A)

Fewer people receive a traditiorral education

stressing verbal and quantitative skiils norv


(Il

(Cl
(

{))

(!l)

than did 20 years ago.


Facility in operating machines designed for use
by nonexperts is almost never enhanced by
expert knowledge of the machines' inner
rvorkings.
N{ost jobs in tomorrorv's job market

will not

demand the ability to operate many machines


that afe designed for use only by experts.
Students cannot attain technological expertise
and also receive an education that does not
neglect verbal and quantitative skills.
When learning to use a machine, technological
expertise is never more important than verbal
and quantitative skills.

Sources: PrcpTest

37, Section 2, Question 75

Preplest 36, Sectlon 3, Question 72


PrepTest 37, Section 4, Questlon 75
Preplest 36, Sectlon 3, Questlon 74

214

I(APLAN

PDF compression, OCR, web optimization using a watermarked evaluation copy of CVISION PDFCompressor

Assumption Questions
58.

Herpetologist: Somepsychologistsattribute
complex reasoning to reptiles, claiming that
simple stimulus-response explanations of
some reptiles'behaviors, such as food
gathering, cannot account fbr the complexity
of such behavior. But sincc experiments show
that reptiles are incapable of making major
alterations in their behavior, for example,
when faced u'ith significant changes in their
environiilcnt, these animals must be incapabie
of complex reasonirrg.

that manufacture telecommunication equipment


such as telephones and fax machines have argued
that exportg of their products should be exempted
from the ban,.on the grounds that it is impossible for
a country to remain repressive ra'hen
telecommunication equipment is widely availabie to
the population of that country.

Which one of the following is an assumption on


which the argument given by the manufacturers

required by the herpetologist's argument?

depends?

{A)

(A)

/1-\

Animals could make major changes in t}reir


behavior only if they rvere capable of complex
reasoning.
Simple stimulus-response explanations can in
principle account for all reptile behaviors.
Reptile behavior appears more complex in the
field than laboratory experiments reveal it to
be.

(t))
i

I
(E)

59.

In order to pressure the government of Country S to


become less repressive, some legislators in Country R
v/ant to ban all exports from R to S. Companies in R

Which one of the following is an assumption

ilJ)

60.

(Il1

(C)

If reptiles were capable of complex reasoning,


they would sometimes be able to make major
changes in their behavior.
Complex reasoning and responses to stimuli
cannot both contribute to the same behavior.

common condition
among mountain climbers, and one from which most
people can recover, is caused by the characteristic
shortage of orygen in the atmosphere at high
altitudes. Cerebral edema, a rarer disruption of blood
circulation in the brain that quickly becomes lifethreatening if not correctly treated from its onset,
can also be caused by a shortage of orygen. Since the
symptoms of cerebral edema gesemble those of

Ordinary mountain sickness,

(D

rE\

The government of S has recently increased


the amount of telecomraurricatidn equipment __,,[it allorvs to be imported into the country. )
The telccommunication equipment that ivould
b.e imported into S if the exemption were to frA
be granted rvould not be available solely to

top government officials in S.


A majority of the members of R's legislature
do not favor exempting telecommunication
equipment from the ban on exports to

Country S.
Of all exports that could be sent to Country

S,

telecommunication equipment would be the


most effective in helping citizens of S oppose
that country's repressive government.

Without pressure from Country R, the


government of S would be able to continue
repressing its citizens indefinitely.

oe
II
.||

g
n
o

ol
El

o
3
lrt
5

o(I

ordinary mountain sicknesslcerebral edema is


rspet:ialiy dangerous at high )iltitudes.)
\tiirich one of the following is an assumption on
rvliich tlie argument depends?
(A; 'fhe treatment for ordinary mountain sickness
differs from the treatment for cerebral edema.
(ti; Cerebral edema can cause those who suf[er
from it to slip into a coma within a few hours.
i{l) Unlike cerebral edema, ordinary mouritain
sicknebs involves no disruption of blood
circulation in the brain.
Shortage of oxygen at extremely high altitudes
r l.l )
is likely to affect thinking processes and cause
errors of judgment.
(E) Most people who suffer from ordinary

mountain sickness recover without any


special treatment.

Sources: PrcpTest 36, Sectlon


PrcpTest 29, Sectlon
Preplest 27, Sectlon

3, Quesilon 76

t, Questlon 15
l, euesilon 2I
I(API.AN) LIJ
'IE

PDF compression, OCR, web optimization using a watermarked evaluation copy of CVISION PDFCompressor

LSAT

Mastery Pradice

'{1. Charles:

During recessions unemployment


typically rises. Thus, during a recession air
, pollution due to automobile exhaust decreases,
silce fewer people commute in cars to jobs and
so cars emitting pollutants into the ait are

Alan:

Government subsidies have been proposed in


Cariana to encourage farmers in Rochelle, the
country's principal agricultural region, to
irnplement certain new farming techniques,
Urrless these techniques are implemented,
erosion of productive topsoil cannot be
coritrolled. Unfortunately, farmers cannot afford
to shoulder the entire cost of the new
tc,:hniques, which are more expensive than those

used less.

Darlar

Why think that air pollution would

decrease?

During

a recession

fewer people can

afford to buy new cars, and cars tend to emit


'more pollutants as they get older.

cnrrentiy

lVhich one of the following is an assumption on


r,,hich Charles's argument depends?
tA

l;
)^(
)^(

r
I'

!0
F
.I

a-

-r

o
UI

ru

o
g,
I

form of public transportation to commute to


their jobs.
During a recession, decreases in the use of cars
resulting from reductions in commuting to
iobs are not oft'set by increased use of cars for
other reasons.
During a recessiotr, a higher proportion of
people who commute in cars to their jobs lose
those jobs than do people who do not use cars
to commute to their jobs.

lrithout

subsidies,

Bettl'r

IJii'i er..osion in Rochelle is caused by


lecuir'iiig flooris, r'.'hich will end next year once

Cariana corrrplr:tes the hydroelectric dam it is


iruilding ;:-: o:;., the region's major river.
'fhereforr, il.r: irelle's total agricultural output
lvill stabi:i2., ,-i its present level even without

ubsidie:..

1',;,r"i,

r'.

(
:

r rre

of tire f ,:i owing is an assumption on

'i li' tty's argulilent


i,riilding

i.

,' ,

tu

II

Therefore,

output in Rochelle rvill inevitablv

]:,:l;.1:'

People who have never been employed drive


no less frequently during a recession than

they rvould othenvise'


Most air pollution is caused by automobile
exhaust emitted by cars used by people
commuting to jobs.
idost people who are employed do not use any

r-rsed.

.,*t

depends?

dam across Rochellds major river will


l,ut reduce any recurrent flooding that occurs
'r regions of Cariana other than Rochelle.
, iie new farming techniques that must be
i'nplemented to control soil erosion in Rochelle
.rre not rvell suited to other regions of Cariana.
a

current yearly output, if any, from


ll.ochelie's land that will be permanently
,rnder lvater once the dam is completed will at
Jcast be matched by additional yearly output

i ire

trom Rochelle's remaining land.

l-he cost to the government of Cariana to

uo

o
J

operate the hydroelectric dam will not be


greater than the projected cost of subsidizing
the farmers of Rochelle in the implementation
of the new farming techniques.
'lhe government of Cariana has sufficient
financial resources both to subsidize its
farmers' implementation of new farming
techniques and to operate a hydroelectric dam,

Sources: fuepTest
PrepTest

216

22, Section 2, Question !4


2!, Sectlon 2, Question 71

I(APLAN

PDF compression, OCR, web optimization using a watermarked evaluation copy of CVISION PDFCompressor

Assumption Quesfions
63.

By examining fossilized beetles, a research team has


produced the most detailed description yet of
temperatures in Britain over the past 22'0fi) years.
Fossils of species that still exist were selected and
dated. When individuals of several species found in
the same place were found to date to the same
. period, the known temPerature tolerances of the
species were used to determine the
, i*itting beetle
maximum summer temperature that could have
existed at that place and Period.
The procedure of the researchers assumes which one

of the following?

(A)
(R)
it-l)
iI))

(E)

Beetles can tolerate warm weather better than

cold weather.
Fossils of different species found in the same
place belonged to different period.
The process of dating is more accurate for
beetles than for other organisms.
The highest actual summer temPerature at a
place and period equaled the average of the"
highest temPeraturs that could have been
tolerated by each of the beetle species found
there and dated to that Period'
The temperature tolerances of the beetle
species did not change significantly during

65. A university should not be entitled to patent the


inventions of its faculty members. Universities, as
guarantors of intellectual freedom, should encourage
the free flow of ideas and the general dissemination
of knowledge. Yet a university that retains the right
to patent the inventions cf its faculty members has a
motive to suppress inforrnation about a potntially
valuable discovery until the patent for it has been
secured, Clearly, suppressing information concerning
such discoveries is incompatible with the university's

obligation to promote the free flow of


arg,ument makes?

(A)
(B)
(C)
D)
iii

is likely that many plant species that are


now extinct could have provided us with
substances that rvould have been a boon to
humanity. Therefore, if we want to ensure that
chemicals from plants are available for use in the
species,

it

future,l4e must make more serious efforts to


preserve for all time our natural rebources'

Y: But living things

are not our "resources." Yours is


a selfish approach to conservation, We should
rather strive to preserve living species because
they deserve to,survive, not because of the good

they can do us'


X's argument relies on which one of the following
assumptions?

iA)
(B)
(C)
(D)
(E)

Medicine would now be more advanced than it


is if there had been a serious conservation
policy in the past.
All living things exist to serve humankind.
The use ofrare and endangered plant species
as a source for chemicals will not itself render
those species extinct.
The only way to persuade people to preserve
natural re$ources is to convince them that it is
in their interest to do so.
Fes if any, plant species have been saved from
extinction through human efforts.

Universities are the only institutions that have


an obligation to guarantee intelleaual freedom.
Most inventions by university faculty members
wouid be profitable if patented.

Publication of reports on research is the only


pra,.:tical way to disseminate information
crrncerning nerv discoveries.

Uriversities that have a motiv to suppress


information concerning discoveries by their
l:,rculty members will occasionally act on that
motive.
If the inventions of a university faculty
member are not patented by that university,
then they will be patented by the faculty
member instead.

64, , X: Since many chemicals useful for agriculture and

Which one of the following is an assumption that the

the 22,000-year Period.

medicine derive from rdre or endangered plant

ideas.

Impact craters caused by meteorites smashing into


Earth have been found all around the globe, but they
have been found in the greatest density in
geologically stable regions. This relatively greater
abundance of securely identified craters in
geologically stable regions must be explained by the
lower rates ofdestructive geophysical processes in
those regions.

*
*
*

o
tl

ce
II
EL

n
o
gl

tl

=.
=
oe

The conclusion is properly drawn if which one of the


following is assumed?

(A)
(B)
(C)
(D)
(E)

A meteorite that strikes exactly the same spot


as an earlier meteorite will obliterate all traces
of the earlier impact.
Rates of destructive geophysical processes
within any given region vary markedly
throughout geological time.
The rate at which the Earth is struck by
meteorits has greatly increased in
geologically recent times.
Actual meteorite impacts have been scattered
fairly evenly over the Earth's surface in the
'course of Eartht geological history.
The Earth's geologically stable regions have
been studied more intensively by geologists
than have its less stable regions.

17, Sectlon 8, Questton 25


Prcpleil 70, Sectlon 4, Quesuon 8
Preplest 9, Sectlon 2, Quesilon 19
Prcplcst 5, Sectlon I, euesilon lll

Sources: Prcplaat

r(APrA9 217

PDF compression, OCR, web optimization using a watermarked evaluation copy of CVISION PDFCompressor

ISAT Mastery

67.

Pradice

Being articulate has been equated with having alatge


vocabular,v. Actually, however, people r,r'ith large
vocabularies have no incentive for, and tend not to
engage in, the kind of creative linguistic selfexpression that is requrired when no available words
seem adequate. Thus a large vocabulary is a
hindrance to using language in a truly articulate way.

Which one of the following is an assumption made

il

the *rgument?

(A)
iBi
'
{C)

*
*
*

(U;
(E)

bo
g
t:

tr

l4'l-ren people are truly articulate, they have the


capacity to express themselves in situations in
which their vocabularies seem inadequate'
People who are able to express themselves
creatively in new situations have little

incentive to acquire large vocabularies.


The most articulate people are people who
have large vocabularies but also are able to
express themselves creatively when the
situation demands it.
In educating people to be more articulate, it
r.;'ould be futile to try to increase the size of
their vocabularies.
ln unfamiliar situations, even people with
large vocabularies often do not have
specifically suitable words available'

a
ul

rg

c,

*
-IE

T-

bo

o
J

Soutce,' Preplest 5, Sectlon

.)
L

1tv

3, Questlon

t7

Q I(APIAN)
.-___/

PDF compression, OCR, web optimization using a watermarked evaluation copy of CVISION PDFCompressor

Assumption Questions
68.

The desire for praise is the desire to obtain, as a sign


that one is good, the favorable opinions of others, But
because people merit praise only for those actions
motivated by a desire to help others, it follows that
one who aids others primarily out of a desire for
praise does not deserve praise for that aid.

Which one of the following, if assumed, enables the


conclusion of the argument to be properly drawn?

(A)
(B)
(C)
(D)
(E)

An action that is motivated by a desire for the


favorable opinion of others cannot also be
motivated by a desire to help others.
No action is worthy of praise if it is motivated
solely by a desire for praise.
People who are indifferent to the welfare of
others do not deserve praise.
One deserves praise for advancing one's olvn
interests only if one also advances the
interests ofothers.
is the motives rather tllan the consequences
of one's actions that determine whether one
'deserves praise for them.

It

70.

Large-scale government projects designed to benefit


everyone-such as roads, schools, and bridgesusually benefit some small segments of society,
initially at least, more than others. The more

equally and widely political power is distributed


among the citizenry, the less likely such projects
are to receive funding. Hence, government by
referendum rather than by means of elected
representatives tends to diminish, not enhance,
the welfare of a society.

Which one of the following is an assumption on


which the argument depends?

(A)
(B)
(C)
(D)
(E)

69.

Physician: Hatha yoga is a powerful tool for helping


people quit smoking. In a clinical trial, those who
practiced hatha yoga for 75 minutes once a week and
received individual counseling reduced their smoking
and cravings for tobacco as much as did those who
went to traditional self-help groups once a week and
had individual counseling.

Which one of the following is an assumption on


which the physician's argument relies?

(A)
(B)
(C)
(D)
(E)

The individual counseling received by the


smokers in the clinical trial who practiced
hatha yoga did not help them quit smoking.
Most smokers are able to practice hatha yoga

more than once

a week.

Traditional self-help groups are powerfril tools


for helping people quit smoking.
People who practice hatha yoga for 75 minutes
once a week are not damaging themselves
physically.

Other forms of yoga are less effective than


hatha yoga in helping people quit smoking.

Large-scale government projects sometimes


enhance the welfare of societv,
Large-scale projects are more likely to fulfill
their intended purposes if they are not
executed by the government.

Government by referendum actually


undermines the democratic process.
The primary purpose of an equal distribution
of political power is to enhance the welfare of
society.

Government by referendum is the only way to


distribute political power equally and widely.

71. Moderate exercise lowers the risk of blockage of the


arteries due to blood clots, since anything that lowers
blood cholesterol levels also lowers the risk of
hardening of the arteries, which in turn lowers the
risk of arterial blockage due to blood clots; and, if the
data reported in a recent study are correct, moderate
exercise lowers blood cholesterol levels.

The conclusion drawn above follows logically


which one of the following is assumed?

(A)
(B)
(C)
(D)
(E)

if

The recent study investigated the relationship


between exercise and blood cholesterol levels.
Blockage ofthe arteries due to blood clots can
be prevented.
Lowering blood cholesterol levels lowers the
risk of blockage of the arteries.
The data reported in the recent study are
correct.
Hardening of the arteries increases the risk of
blockage ofthe arteries due to blood clots.

Sources,' PrcpTost 37, Sectlon


PrepTeat 37, teetion
Preplest 36, Sectlon

*
*
*
*

E.
fr

E
7
o
ql
lrf

o
l!.
3

oq

2, Question t9
4, Questions t9 and 2O

!,

Questlon

!8

{tP

21e

PDF compression, OCR, web optimization using a watermarked evaluation copy of \CVISION PDFCompressor

In the paintings by seventeenth-century Dutch artist


Vermeer, we find several recutrent items: a satin
iacket, a certain Turkish carpet, and wooden chairs
with lion's head finials. These reappearing objects
might seem to evince a dearth of props. Yet we know
that many of the props Vermeer used were expensive'
Thus, while we might speculate about exactly why
Vermeer worked with a small number of familiar
objects, it was clearly not for lack of props that the
recurrent items were used'

74. It is said that people should accept themselves as they


are instead of being dissatisfied with their own
abilities. But this is clearly a bad principle if the goal
is a society whose citizens are gertuinely happy,

Which one of the following is an assumption


required try the argument?

(A)

The conclusion follows logically if which one of the


following is assumed?

(A)

*
*
*
*

(B)
(C)
(D)
(E)

UE
II

tr

8 'r.

s@

!o
.E@
"u0
,Ol

The conclusion can be properly inferred if which one


of the following is assumed?

(A)

Writers who sometimes use words


ambiguously have no readers who try to
attain a precise understanding of what the

(B)

Writers whose purpose is personal expression


are unioncerned with tvhether anyone enjoys

(C)
(D)
(E)

(D)

sister.

Any writer whose purpose is personal expression


sometimes uses words ambiguously. Every poet's
purpose is personal expression. Thus no Poetry
reader's enjoyment depends on attaining a precise
understanding of what the poet means.

,ill

(C)

The several lecurrent items that appeared in


Vermeer's paintings had special sentimental

recurrent objects in Vermeer's paintings' we


would not see expensive props in any of them.

fi

(B)

Vermeer often borrowed the expensive props


he represented in his paintings.
The props that recur in Vermeer's paintings
were always available to him.
The satin jacket and wooden chairs that recur
in the paintings were owned byVermeer's

importance for him.


Ifa dearth ofprops accounted for the

writer means.
reading their works.
No writer who ever uses words ambiguously
has any reader whose enjoyment depends on
attaining a precise understanding of what the

for

no one can be genuinely happy if he or she is not


pursuing personal excellence and is unwilling to
undergo personal change of any kind.

(E)
/J,

Those who are willing to change will probably


find genuine happiness.
People who are not dissatisfied with
themselves are less likely than others to
pursue personal excellence.
Personal excellence cannot be acquired by
those who lack genuine confidence in their

own abilities.
People are justified in feeling content with
themselves when they have achieved some
degree of personal excellence.
Happiness is not genuine unless it is based on
something that is painftrl to obtain.

The current pattern of human consumption of


resources, in which we rely on nonrenewable
resources, for example metal ore, must eventually
change. Since there is only so much metal ore
available, ultimately we must either do without or
turn to renewable resources to take its place.

Which one of the following is an assumption


required by the argument?

(A)
(B)
(C)
(D)
(E)

There are renewable resource replacements for


all of the nonrenewable resources currently
being consumed.
We cannot indefinitely replace exhausted
nonrenewable resources with other
nonrenewable resources.
A renewable resource cannot be exhausted by

human consumption.
Consumption of nonrenewable resources will
not continue to increase in the near future.
Ultimately we cannot do without
nonrenewable resources.

writer means.
Most writers whose readers'enjoyment does
not depend on attaining a Precise
understanding of the writers'words are poet$.
Readers who have a precise understanding of
what a writer has written derive their
enjoyment from that understanding.

Sources: Prepfest 36, Section 7, Questlons 26 and 22


PrepTest 36, Section 3, Questlon 78
Prcpfest 35, Sactlon !, Queetlon !8

220

I(APtAN

PDF compression, OCR, web optimization using a watermarked evaluation copy of CVISION PDFCompressor

Assumption Questions
can be an expert, for
there are no of6cial guidelines determining
what an expert must know. Anybody who
manages to convince some people of his or her
qualifications in an area-whatever those may
be-is an expert.

Columnist: Almost anyone

78.

environmental problems, for while the ease of


attributing feelings to large mammals facilitates
evoking sympathy for them, it is more difficult to
elicit sympathy for other kinds of organisms, such as
the soil microorganisms on which large ecosystems
and agriculture depend.

The columnist's conclusion follows logically if which


one of the following is assumed?

(A)
(B)
(C)
(D)
(E)

Publicity campaigns for endangered species are


unlikely to have much impact on the most important

Which one of the following is an assumption on


which the argument depends?

Almost anyone can convince some people of


his or her qualifications in some area.
Some experts convince everyone of their
qualifications in almost every area.
Convincing certain people that one is qualified
in an area requires that one actually be
qualified in that area.
Every expert has convinced some people of his
or her qualifications in some area'
Some people manage to convince almost
everyone of their qualifications in one or

(A)
(B)
(C)
(D)

more areas,
77.

@
@

(E)

No chordates are tracheophytes, and all members of


Pteropsida are tracheophyles' So no members of
Pteropsida belong to the family Hominidae.
The conclusion above follows logically if which one
of the following is assumed?

(A)

All members of the family Hominadae are

(B)

All members of the family Hominidae are

(c)
(D)

All tracheophytes are members of Pteropsida.


No members of the family Hominidae are

(E)

No chordates are members of Pteropsida.

tracheophytes.
chordates.

chordates.

79.

The most important environmental problems


involve endangered species other than large
mammals.
Microorganisms cannot experience pain or
have other feelings.
Publicity campaigns for the environment are
the most effective when they elicit sympathy
for some organism.
People ignore environmental problems unless
they believe the problems will affect creatures
with which they sympathize.
An organism can be environmentally significant
only if it affects large ecosystems or agriculture.

*
*
*
*

Several critics have claimedthat any contemporary

poet who writes formal poetry-poetry that is


rhymed and metered-is performing a politically
conservative act. This is plainly false. Consider Molly
Peacock and Marilyn Hacker, two contemporary poets
whose poetry is almost exclusively formal and yet who
are themselves politically progressive feminists.
The conclusion drawn above follows logically
which one of the following is assumed?

if

(A)

No one who is a feminist is also politically

(B)

No poet who writes unrhymed or unmetered


poetry is politically conservative.
No one who is politically progressive is capable
of performing a politically conservative act.
Anyone who sometimes writes poetry that is
not politically conservative never writes
poetry that is politically conservative.
The content of a poet's work, not the work s
form, is the most decisive factor in
determining what political consequences, if
any, the work will have.

(C)
(D)
(E)

conservative.

Sources: PrepTest
PrepTest
PrepTest
PrepTest

!, Questlon 2O
t, Questlon 22
35, Sectlon 4, Questlon t8

35, Sectlon

35, Sectlon
35,

S*tlon 4, Questlon 79

f$ry

221

PDF compression, OCR, web optimization using a watermarked evaluation copy of CVISION PDFCompressor

LSAT

Mostery Practice

80. Ethicist: Studies have documented

the capacity

of

placebos to reduce pain in patients who believe


that they are receiving beneficial drugs. Some
doctors say that they administer placebos
because medically effective treatment reinforced
by the placebo effect sometimes helps patients
recover faster than good treatment alone. But
administering placebos is nonetheless ethically
questionable, for even if a placebo benefits a
patient, a doctor might, for example, have
prescribed it just to give the patient satisfaction

that something was being done.

*r
*
*
*

The ethicist's argument depends on which one of the


following assumptions?

(A)
(B)
(C)

bo
rrtr

(D)

e
a
tu
go

(E)

a-

ru

A patient's psychological satisfaction is not a


consideration in administering medical
treatment.
The motivation for administering a placebo
can be relevant to the ethical justification for
doing so.
Medical treatment that relies on the placebo
effect alone is ethically indefensible.
The pain relief produced by the placebo effect
justifies the deception involved in
administering a placebo.
Administering a placebo is not ethically
justified if that treatment is not prescribed by
a doctor,

81.

The price of a firll-fare coach ticket from Toronto to


Dallas on Breezeway Airlines is the same today as it
was a year ago, if inflation is taken into account by
calculating prices in constant dollars, However, today
90 percent of the Toronto-to-Dallas coach tickets that
Breezeway sells are discount tickets and only 10
percent are fi.rll-fare tickets, rvhereas a year ago half
were discount tickets and half were fuIl-fare tickets.
Therefore, on average, people pay less today in
constant dollars for a Breezeway Toronto-to-Dallas
coach ticket than they did a year ago.

Which one of the following, if assumed, would allow


the conclusion above to be properly drawn?

(A)
(B)
(C)

A Toronto-to-Dallas full-fare coach ticket on


Breezeway Airlines provides ticket-holders
with a lower level of service today than such
ticket provided a year ago.
A Toronto-to-Dallas discount coach ticket on
Breezeway Airlines costs about the same
amount in constant dollars today as it did a
year ago.

All full-fare coach tickets on

Breezeway

Airlines cost the same in constant dollars

(D)
(E)

as

they did a year ago.


The average number of coach passengers per
flight that Breezeway Airlines carries from
Toronto to Dallas today is higher than the
average number per flight a year ago.
The criteria that Breezeway Airlines uses for
permitting passengers to buy discount coach
tickets on the Toronto-to-Dallas route are
different today than they were a year ago.

.g

bto

o
J

Sources.' PrepTest 33, Sectrbn


PtepTest 29, Sectlon

!,

Questlons

t3

7, Questlon 2O

222

PDF compression, OCR, web optimization using a watermarked evaluation copy of CVISION PDFCompressor

82. Historian:

We can learn about the medical history


of individuals through chemical analysis of
their hair. It is likely' for example, that Isaac
Newton's psychological problems were due to
mercury poisoning; traces of mercury were
found in his hair. Analysis is now being done
on a lock of Beethoven's hair. Although no
convincing argument has shown that,
Beethoven ever had a venereal disease, some
people hypothesize that venereal disease
caused his deafness. Since mercury was
commonly ingested in Beethoven's time to
treat venereal disease, if researchers find a trace
of mercury in his hair, we can conclude that
this hypothesis is correct,

83. Monica:

The sculpture commissioned for our town


plaza has been scorned by the public ever since
it went up. But since the people in our town do

not know very much about contemporary art,


the unpopularity of the work says nothing
about its artistic merit and thus gives no
reason for removing it.

Hector: You may be right about what the


sculpture's popularity means about its artistic
merit. However. a work of art that was
commissioned for a public space ought to
benefit the public, and popular opinion is
ultimately the only way of determining what
the public feels is to its benefit. Thus, if public

opinion of this sculpture is what you say, then

it certainly ought to

Which one of the following is an assumption on


which the historian's argument depends?

(A)

None of the mercury introduced into the body

(B)

can be eliminated.
Some people in Beethoverls time did not

(C)

ingest mercury.
Mercury is an effective treatment for venereal

(D)

Mercury poisoning can cause deafness in

(E)

people with venereal disease.


Beethoven suffered from psychological
problems of the same severity as Newtont.

be removed.

The argument Hector makes in responding to


Monica depends on the assumption that

(A)

disease.

(B)

(c)
(D)
(E)

84.

no matter what the public's opinion is on an


issue affecting the public good, that public
opinion ought to be acted on, eyell though
the opinion may not be a knowledgeable one
Monica's assessment of the public's opinion of
the sculpture is accurate
if the sculpture had artistic merit, then even a
public that was not knowledgeable about
modern art would not scorn the sculpture
worlis of art commissioned for public spaces
ought not to be expected to have artistic merit
if the public feels that it does not benefit from
the sculpture, this shows that the public does
not in fact benefit from the sculpture

The dwarf masked owl, a rare migratory bird of prey,


normally makes its winter home on the Baja
peninsula, where it nests in the spiny cactus. In fact,
there are no other suitable nesting sites for the dwarf
masked owl on the Baja peninsula. But a blight last
spring destroyed all of the spiny cacti on the Baja
peninsula. So unless steps are taken to reestablish the
spiny cactus population, the dwarf masked owl will
not make its home on the Baja peninsula this winter.
The argument depends on assuming which one

*
*
*
*

q3.

l'l

g
vo
Ol

{fi

o
:I
=.

oq

of

the following?

(A)
(B)
(C)
(D)
Sowces: PrcpTest 28, Sectlon 3, Questlon 76
fueplest 26, Sectlon 2, Questlon 25
Preplest 25, Sectlon 4, Question 25

(E)

No birds of prey other than the dwad masked


owl nest in the spiny cactus.
If the Baja peninsula contains spiny cacti, then
the dwarf masked owl makes its winter honre
there.

On occasion the dwarf masked owl has been


known to make its lvinter bome far from its
normal migratory route.
The dwarf masked orvl will not make its winter
home on the Baja peninsula only if that
region contains no spiny cacti.
Suitable nesting sites must be present where
the dwarf masked owl makes its winter home.
I(APLAN

223

PDF compression, OCR, web optimization using a watermarked evaluation copy of CVISION PDFCompressor

The widespread staff reductions in a certain region's


economy are said to be causing people whostill have
their jobs to cut back on ne\{i purchases as though
they, ioo, had become economically distressed'
Clearly, however, actual spending by such people is
undiminished, because there has been no unusual
increase in the amount of money held by those
people in savings account'

E5.

(C)
(D)

b0

(E)

,S

rT

tt
tg

{9,

ru

rJ

r:

!o

Which one of the following is an assumption on


which the argument relies?
(A) If people in the region who continue to be
emplbyed have debts' they are not now paying
them off at an accelerated rate.
(B) People in the region who continue to be
employed and who have relatlves who have
losi their jobs commonly assist those relatives

*
*
*
*

sE

87.

86.

Each of the following is an assumption on which

Professor Robinson' s argument depends EXCEPT:

(A)

The crater indicates an impact of more than


sufficient size to have caused the mass

(B)

The recovered rocks recrystallized shortly after


they melted.
No other event caused the rocks to melt after
the imPact formed the crater.
The recovered rocks nelted as a result of the
impact that formed the crater.
The mass extinction would have occurred soon
after the impact that supposedly caused it.

(D)
(E)

(A)

Having similar characteristics is not a sign that


types of animals are evolutionarily related.
Dromeosaurs and birds could have common

(B)

ancestors .
Knowledge of dromeosaur fossils and the
earliest bird fossils is complete.
Known fossils indicate the relative dates of
origin of birds and dromeosaurs.
Dromeosaurs are dissimilar to birds in many

(C)
(D)

than the ones theY lost.


People in the region who continue to be

Professor Robinson: A large meteorite impact


crater in a certain region was thought to be the
clue to explaining the mass extinction of plant
and animal species that occurred at the end of
the Mesozoic era' However, the crystalline
structure of rocks recoveted at the site indicates
that the impact that formed this crater luas not
the culprit. When molten rocks crystallize'
they display the polarity of Earth's magnetic
field at thai time' But the recrystallized rocls
recovered at the site display normal magnetic
polarity, even though Earth's magnetic field
was reversed at the time of the mass extinction'

(C)

The expert's argument depends on assuming which


one of the following?

financiallY.
If people in the region who have lost jobs get
nirv iobs, the new jobs generally pay less well
employed are pessimistic about thei'r
prosPects for increasing their incomes'
fhere exist no statistics about sales ofgoods in
the region as a whole.

erpert: Some paleontologists have


claimed that birds are descendants of a group
of dinosaurs called dromeosaurs. They appeal
to the fossil record, which indicates that
dromeosaurs have characteristics more similar
to birds than do most dinosaurs. But there is a
fatal flaw in their argument; the earliest bird
fossils that have been discovered date back tens
of millions of years farther than the oldest
known dromeosaur fossils. Thus the
paleontologists' claim is false'

Dinosaur

(E)

significant ways.
88.

Spectroscopic analysis has revealed the existence of


frozen nitrogen, methane, and carbon monoxide on
the surface of Pluto. Such ices have a tendency to
vaporize, producing an atmosphere. Since the
proportion of any gas in such an atmosphere
depends directly on how readily the corresponding
ice vaporizes, astronomers have concluded that the
components of Pluto's atmosphere are nitrogen,
carbon monoxide, and methane, in order of
decreasing abundance.
The astronomers'argument relies on which one
the followin g assumptions?

(A)
(B)
(C)

extinction.

(D)
(E)

of

There is no more frozen nitrogen on the


surface of Pluto than there is either frozen
carbon monoxide or methane.
Until space probes reach Pluto, direct analysis
of the atmosphere is impossible.
There is no frozen substance on the surface of
Pluto that vaporizes more readily than
methane but less readily than carbon

monoxide.
Nitrogen is found in the atmosphere of a
planet only if nitrogen ice is found on the
surface of that planet.
A mixture of nitrogen, carbon monoxide, and
methane is characteristic of the substances
from which the Solar System formed.

Sources: PrcpTest 24,9ectlon 3, Questlon 78


fuepTest

22, Sectlon 2, Questlon

t9

Preplest 22, Sectlon 4, Quastlan 22


PrepTest

21, Sectlon 2, Questlon 19

224

PDF compression, OCR, web optimization using a watermarked evaluation copy of CVISION PDFCompressor

Assumption Questions
89.

Eva:

A "smart highway" system should be installed,


one that would monitor areawide traffic

patterns and communicate with computers in


vehicles or with programmable highway signs
to give drivers information about traffic
congestion and alternate routes. Such a system,

91.

patron: The apples sold in this cafeteria


are greasy. The cashier told me that the apples
are in that condition when they are delivered
to the cafeteria and that the cafeteria does not
wash the apples it sells. Most fruit is sprayed
with dangerous pesticides before it is
harvested, and is dangerous until it is washed.
Clearly, the cafeteria is selling pesticide-

Cafeteria

we can infer, would result in improved traffic

flow in and around cities that would do more


than improve drivers'tempersi it would
decrease the considerable loss of money and
productivity that now results from traffic

covered fruit, thereby endangering its patrons.

Which one of the following is an assumption on


which the argument depends?

congestion.

Luis:

(A)

There are already traffic rePorts on the radio.


Why would a "smart highway''system be any
better?

(B)

Eva's argument depends on the assumption that

(A)
(B)
(C)
(D)
(E)

(C)

on "smart highways" there would not be the


breakdowns of vehicles that currently cause
traffic congestion
trafEc lights, if coordinated by the system,

would assure a free flow of traffic


traffic flow in and around cities is not now so
congested that significant improvement is
impossible
the type of equipment used in "smart highway''
systems would vary from one city to another
older vehicles could not be fitted with
equipment to receive signals sent by a "smart
highway" system

Ann will either take a leave of absence from


Technocomp and return in a year or else she will quit
her job there; but she would not do either one unless
she were offered a one-year teaching fellowship at a
prestigious university. Technocomp will allow her to
take a leave of absence if it does not find out that she
has been offered the fellowship, but not otherwise.
Therefore, Ann will quit her job at Technocomp only
if Technocomp finds out she has been offered the
fellowship.

(D)
(E)
92.

Nuclear reactors are sometimes built in "geologically


quiet" regions, so called by geologists because such
regions are distant from plate boundaries and
contain only minor faults. Since no minor fault in a
geologically quiet region produces an earthquake
more often than once in any given 100,000-year
period, it follows that of all potentiai nuclear reactor
sites in such a region, those that are least likely to be

struckby an earthquake are ones located near a fault


that has produced an earthquake within living
memory.

(A)
(B)
(C)
(D)
(E)

Technocomp will find out about Ann being


offered the fellowship only if someone
informs on her.
The reason Ann wants the fellowship is so she
can quit her job at Technocomp.
Technocomp does not allow any of its
employees to take a leave of absence in order
to work for one of its competitors,
Ann will take a leave of absence if Technocomp
allows her to take a leave of absence.
Ann would be offered the fellowship only if
she quit her job at Technocomp.

oe
IT
rlt
ql
I

!f
t,|

ort

which the argument depends?

(A)

Geologically quiet regions are the least


dangerous regions in which to build nuclear

(B)

For any potential nuclear reactor site, the


likelihood of being struck by an earthquake is
the primary determinant of site safety.
In a geologically quiet region, every potential

(C)

*
*
*
*

TI
=
I
)

Which one of the following is an assumption on

Which one of the following, if assumed, allows the


conclusion above to be properly drawn?

The apples that the cafeteria sells are not


thoroughly washed after harvest but before
reaching the cafeteria,
Most pesticides that are sprayed on fruit before
harvest leave a greasy residue on the fruit.
Many of the cafeteria's patrons are unaware that
the cafeteria does not wash the apples it sells.
Only pesticides that leave a greasy residue on
fruit can be washed off.
Fruits other than apples also arrive at the
cafeteria in a greasy condition.

reactors.

nuclear reactor site is near at least one minor

(D)

(E)

fault.
Nuclear reactors that are located in
geologically quiet regions are built to
withstand at least one but not necessarily
more than one earthquake of minor to
moderate force.
Earthquake faults in geologically quiet regions
produce earthquakes at least once in 100,000
years.

Sources; PrcpTest 21, Sectlon 3, Questlon


PrepTest 27, Sectlon 2, Quastlon 20
Prcplest 17, Sectlon 2, Questlone t0 and 27

22s
PDF compression, OCR, web optimization using a watermarked evaluation copy of CVISION PDFCompressor

ISAT Mostery Practice

93.

Magazine editor: I know that some of our regular


advertisers have been pressuring us to give
favorable mention to their products in our
articles, but they should realize that for us to
yield to their wishes would actually be against
their interests. To remain an effective
advertising vehicle we must have loyal
readership, and we would soon lose that
readership if our readers suspect that our
editorial integrity has been compromised by
pandering to advertisers.

94,

Even the earliest known species of land animals,

known from fossils dating from the late Silurian


period, 400 million years ago, show highly evolved
adaptations to life on land. Since neither aquatic nor
amphibious animals exhibit these adaptations, early
species of land aaimals must have evolved very
rapidly after leaving an aquatic environment.
Which one of the following is an assumption on
which the argument depends?

(A)

director: You underestimate the


sophistication of our readers. They recognize
that the advertisements we carry are not
articles, so their response to the

Advertising-sales

*
*
*
*

advertisements has never depended on their


opinion of the editorial integrity of the
magazine as a whole.

ElD

o
It
G
o

E
I

(B)

o
J

A magazine editor should never be in{luenced


in the performance of his or her professional
duties by the wishes of the companies that
regularly advertise in the magazine.
The magazine cannot give any favorable
mention in its articles to its regular
advertisers without compromising its

(D)

reputation for editorial integrity.


Favorable mention of their products in the
magazine's articles is of less value to the
advertisers than is the continued effectiveness
of the magazine as an advertising vehicle.
Giving favorable mention to a product in a

(B)

magazine article is a more effective form of


advertisement than is an explicit advertisement
for the product in the same magazine,
Carrying paid advertisements can never pose

(C)

IE

tJ
rI
bo

(C)
(D)

The magazine editor's argument assumes which one


of the following?

(A)
F
I
rI

(B)

(E)

Known fossils of early land animals include


fossils of animals that lived relatively soon
after the first emergence of land animals.
Fossils from the late Silurian period represent
only a small number of the animal species
that were alive at that time.
No plants were established on land before the
late Silurian period.
No present-day species of aquatic animal is
descended from a species of animal that once
lived on land.
All animals alive in the late Silurian period
lived either exclusively on land or exclusivelv

in the water.
95.

@
@

A poor farmer was fond of telling his children: "In


this world, you are either rich or poor, and you are
either honest or dishonest. All poor farmers are
honest. Therefore, all rich farmers are dishonest."
The farmer's conclusion is properly drawn if the

argument assumes that

(A)
(B)
(C)
(D)
(E)

every honest farmer is poor


every honest person is a farmer
everyone who is dishonest is a rich farmer
eYeryone who is poor is honest
every poor person is a farmer

any threat to the magazine's reputation for


editorial integrity nor to the loyalty of its
readership.

Sources; PrepTest 17, Sectlon 2, Questlon 23

!7, Sectlon 3, Question 2


PrcpTest 9t Sectlon 2, Questlon 23
PrepTest

226
PDF compression, OCR, web optimization using a watermarked evaluation copy of CVISION PDFCompressor

Assumption Questions
96.

Medical research findings are customarily not made


public prior to their publication in a medical journal
that has had them reviewed by a panel of experts in a
process called peer review. It is claimed that this
practice delays public access to potentially beneficial
information that, in extreme instances, could save
lives. Yet prepublication peer review is the only way
to prevent erroneous and therefore potentially
haimful information from reaching a public that is ill
equipped to evaluate medical claims on its own.
Therefore, waiting until a medical journal has
published the research findings that have passed peer
ieview is the price that must be paid to protect the
public from making decisions based on possibly

97. Marcusl

For most ethical dilemmas the journalist is


likely to face, traditional journalistic ethics is
clear, adequate, and essentially correct. For
example, when journalists have uncovered
newsworthy information, they should go to
press with it as soon as possible. No delay

motivated by the journalists'personal or


professional interests is permissible.

Anita:

before the public-that is a journalistt job.


But in the typical case, where a journalist has
some information but is in a quandary about
whether it is yet important or "newsworthy,"
this guidance is inadequate.

substandard research'
The argument assumes that

(A)
(B)
(C)
(D)
(E)

unless medical research findings are brought

to peer review by a medical journal, peer


review will not occur
anyone who does not setve on a medical
review panel does not have the necessary
knowledge and expertise to evaluate medical
research findings
the general public does not have access to the
medical iournals in which research findings
are published
all medical research findings are subjected to
prepublication Peer review
peer review panels are sometimes subject to
political and professional pressures that can
make their judgments less than impartial

Well, Marcus, of course interesting and

important information should be brought

In order to conclude properly from Anita's


statements that Marcus'general claim about
traditional journalistic ethics is incorrect, it would
have to be assumed that

(A)
(B)

whether a piece of information is or is not


newsworthy can raise ethical dilemmas for
journalists
there are circumstances in which it would be
ethically wrong for a journalist to go to press
with legitimately acquired, newsworthy

(C)

the most serious professional dilemmas that a

*
*
*
*

information
journalist is likely to face are not ethical

(D)

dilemmas
there are no ethical dilemmas that

journalist

is likely to face that would not be conclusively

(E)

resolved by an adequate system ofjournalistic


ethics
for a system ofjournalistic ethics to be
adequate it must be able to provide guidance
in every case in which a journalist must make
a professional decision

Sources.' PrepTest
PrepTest

0
gq
II
a

g.

7
o
Of
o
o
=
3
II

oq

9, Sectlon 2, Queetlon 25
9, Section 4, Questlon t9

227

PDF compression, OCR, web optimization using a watermarked evaluation copy of CVISION PDFCompressor

98. Mayor

of Plainwills In order to help the economy of


Plainsville, I am using some of our tax
revenues to help bring a major highway
through the town and thereby attract new
business to Plainsville.

Citizens'group: You must have interests o&er than


our economy in mind. If you were really
interested in helping our economy, you would
instead aUocate the revenues to building a new
business park, since it would bring in twice the
business that your highwaY would.
The argument by the citizens'group relies on which
one of the following assumptions?

*
*
*
*

(A)
(B)
(C)
(D)

BO
II

tr

t:

(E)

Plainsville presendy has no major highways


running through it.
The mayor accePts that a new business park
would bring in more new business than
would the new highway.
The new highway would have no benefits for
Plainsville other than attracting new business.
The mayor is required to get aPproval for all
tax revenue allocation plans from the city
council.
Plainsvillet economy will not be helped unless
a new business park of the sort envisioned by
the citizens'group is built.

{a
GI

o
sl

-ts
l:

tJ
b0

Source: Prepiest

5, Sectlon

t,

Questlon 3

228
PDF compression, OCR, web optimization using a watermarked evaluation copy of CVISION PDFCompressor

También podría gustarte